a 11.3 cm long solenoid contains 895 turns and carries a current of 5.93 a . what is the strength of the magnetic field at the center of this solen

Answers

Answer 1

The strength of the magnetic field at the center of the solenoid is 1.87 millitesla (mT), or 1.87 x 10^-3 T.

To calculate the strength of the magnetic field at the center of the 11.3 cm long solenoid with 895 turns and a current of 5.93 A, we can use the formula:
B = μ₀ * n * I
where B is the magnetic field strength, μ₀ is the permeability of free space (4π x 10^-7 Tm/A), n is the number of turns per unit length, and I is the current.
First, we need to find the number of turns per unit length (n) by dividing the total number of turns (895) by the length of the solenoid (11.3 cm or 0.113 m):
n = 895 / 0.113 = 7921 turns/m
Now we can plug in the values and solve for B:
B = (4π x 10^-7 Tm/A) * 7921 turns/m * 5.93 A
B = 1.87 x 10^-3 T
Therefore, the strength of the magnetic field at the center of the solenoid is 1.87 millitesla (mT), or 1.87 x 10^-3 T. This value is directly proportional to the current flowing through the solenoid and the number of turns per unit length.

To know more about solenoid visit:

https://brainly.com/question/15504705

#SPJ11


Related Questions

an object has a mass of 50kg, a final height of 20m and an initial height of 8m. what is the amount of work done

Answers

amount of work done is 5880 J

Given:

mass of object = 50kg

Final height = 20m

initial height = 8m

To Find:

amount of work done

Solution:

work is done when a force acts upon an object to cause a displacement. You can calculate the energy transferred, or work done, by multiplying the force by the distance moved in the direction of the force.

The work done by gravity is given by the formula,

W = mgh

W = 50 x 9.8 x ( 20-8)

= 5880 J

So the work done is 5880 J

Learn more about Work done here:

https://brainly.com/question/25239010

#SPJ4

A researcher determined the amount of electrical energy entering a neon lighted sign and compared it with the amount of light produced by the sign. The researcher found that the amount of light energy produced was less than the amount of electrical energy used.

Select the best explanation of this observation.

Group of answer choices

a. Some of the energy was destroyed during the conversion.

b. Some energy was created during the conversion.

c. Some of the energy is converted into thermal energy.

d. Not all of the electrical energy entering the neon sign was used.

Answers

The best explanation of the observation of the researcher is that some of the energy is converted into thermal energy (option C).

How does energy transfer?

Energy, according to the law of conservation of energy, cannot be created nor destroyed but transformed.

According to this question, a researcher found that the amount of light energy produced in a neon lighted sign was less than the amount of electrical energy used.

This suggests that some of the energy is converted into thermal energy.

Learn more about energy at: https://brainly.com/question/1932868

#SPJ1

What mass will accelerate at 3 m/s² when a net force of 150 N acts on it?

Answers

Answer:

[tex]\huge\boxed{\sf m = 50\ kg}[/tex]

Explanation:

Given data:

Acceleration = a = 3 m/s²

Force = F = 150 N

Required:

Mass = m = ?

Formula:

F = ma

Solution:

Put the givens in the formula

150 = m (3)

Divide 3 to both sides

150/3 = m

50 kg = m

m = 50 kg

[tex]\rule[225]{225}{2}[/tex]

In a circuit consisting of two lamps connected in series, if the current in one lamp is 1 a, what is the current in the other lamp?

Answers

The current in the second lamp will also be 1amp seeing that the connection is a series connection.

See the explanation bellow.

What is circuit in series?

The simple rule of a series connection is such that for each load, the current flowing is the same and the resistance varies across the loads.

However, the reverse is the case for a parallel circuit connection, where the current varies and the resistance is the same for each circuit element.

Learn more about electrical circuits here:

https://brainly.com/question/2969220

#SPJ1

The reason the jovian planets lost very little of their original atmosphere is due to their?

Answers

The reason for which the jovian planets lost very little of their original atmosphere is due to their large mass.

What are the jovian planets?

The jovian planets are celestial bodies formed by gaseous huge masses of dust instead of rock planes such as, among others, the Earth planet.

These types of planets (jovian planets)  include four major celestial bodies, i.e., Jupiter, Saturn, Uranus, and Neptune.

The jovian planets are characterized to have a thick atmosphere that is composed of different elements such as hydrogen, and methane.

In conclusion, the reason for which the jovian planets lost very little of their original atmosphere is due to their large mass.

Learn more about  the jovian planets here:

https://brainly.com/question/13037269

#SPJ1

A 2.50 × 105 w motor is used for 26.4 s to pull a boat straight toward shore. how far does the boat move toward shore if a force of 4.20 × 104 n is applied by the motor? 157 m 1,590 m 1.11 × 106 m 6.60 × 106 m

Answers

The distance travelled by the boat towards shore if a force of 4.20x10⁴N is applied is 157.14 m.

Power is a unit of measurement for the amount of work completed in a particular period of time; it may be computed using both work and time. Divide the amount of work by the length of time it takes to do the work to find power. The unit of work is the joule. The unit of power measurement is known as joules of work per second.

The power of the motor = 2.50x10⁵ W

                                     = 2.50x10⁵ N m/s

The time it takes to pull the boat = 26.4 s

The force applied  = 4.20x10⁴ N

Using the formula;

                                     [tex]d = \frac{Pt}{F}[/tex]

where,

P = Power

t = Time taken

F = Force applied

d = Distance travelled

The distance traveled by the boat = 2.50x10⁵ N m /s (26.4 s) /(4.20x10⁴ N)                                                

                                                      =157.14 m

Therefore, the distance travelled by the boat if a force of 4.20x10⁴N is applied is 157.14 m.

Learn more about power here:

https://brainly.com/question/4047067

#SPJ4

During a collision with a wall, the velocity of a 0.200-kg ball changes from 20.0 m/s toward the wall to 12.0 m/s away from the wall. If the time the ball was in contact with the wall was 0.0500 s, what was the magnitude of the average force applied to the ball

Answers

The magnitude of average force applied to the ball is 128N.

To find the answer, we need to know about the average force.

What's average force?According to Newton's law, average force= change in momentum/timeHere, change in momentum= 0.2×20-(-0.2×12) = 6.4 kg m/sTime = 0.05sAverage force= 6.4/0.05= 128N

Thus, we can conclude that the average force is 128N.

Learn more about the average force here:

brainly.com/question/18652903

#SPJ4

A drop of oil is floating between two parallel plates with a separation of 8.3 mm. If the mass of the oil drop is 2.7 x 10-15 kg, and its charge is -9.612 x 10-19 C, determine the voltage across the charged plates and determine which plate is positive.

Answers

The voltage across the charged plates is  228.5 V.

Voltage across the plates

V = Fd/q

V = (mgd)/q

where;

m is mass of the oilg is acceleration due to gravityd is the distance of separationq is charge

V = (2.7 x 10⁻¹⁵ x 9.8 x 8.3 x 10⁻³) / (9.612 x 10⁻¹⁹)

V = 228.5 V

Thus, the voltage across the charged plates is  228.5 V.

Learn more about voltage here: https://brainly.com/question/14883923

#SPJ1

Two harmonic waves traveling in opposite directions interfere to produce a standing wave described by y

Answers

The wavelength of the interfering waves is 3.14 m.

Calculation:

The general equation of a standing wave is given by:

y = 2A sin (kx) cos (ωt)                            ......(1)

The given equation represents the standing wave produced by the interference of two harmonic waves:

y = 3 sin (2x) cos 5t                                .......(2)

Comparing equations (1) and (2):

k = 2

We know that,

k = 2π/λ

λ = 2π/k

λ = 2 (3.14)/ 2

λ = 3.14 m

Therefore, the wavelength of the interfering waves is 3.14 m.

I understand the question you are looking for is this:

Two harmonic waves traveling in opposite directions interfere to produce a standing wave described by y = 3 sin (2x) cos 5t where x is in m and t is in s. What is the wavelength of the interfering waves?

Learn more about interfering waves here:

https://brainly.com/question/2910205

#SPJ4

The decomposition of H2O2 is a first-order reaction given by the formula: Rate = k [H2O2]1. How would the reaction rate be affected if the H2O2 concentration were double its original value?

Answers

Answer: The decomposition of H₂O₂ is a first-order reaction. Then, the reaction rate will be 2 times the original rate, if we double its concentration value.

Explanation: To find the answer, we need to know about the decomposition of H₂O₂.

What is the rate of reaction, if we double the concentration of H₂O₂?We have the equation of decomposition of H₂O₂ as,

                              2H₂O₂→ 2 H₂O+ O₂

We have given that, the equation of reaction rate of the first order reaction as,

                              Rate = k [H₂O₂]1

As we know that the equation of reaction rate is,

              Rate = rate constant × concentration of [H₂O₂]

Thus, by comparing both the equations, we get,

         rate constant=k and the concentration of [H₂O₂] = 1[H₂O₂].

Given that the concentration is doubled. Then the expression for rate will be,

                              Rate = 2k [H₂O₂].

Thus, we can conclude that the rate of reaction will be 2 times the initial rate if we double the concentration.

Learn more about the decomposition of H₂O₂ here:

https://brainly.com/question/28047273

#SPJ1

If we double the concentration value, the reaction rate will be twice as fast as before.

In order to determine the solution, we must understand how H2O2 breaks down.

If we double the concentration of H2O2, what will happen to the reaction rate?The formula for the breakdown of H2O2 is,

                        2H₂O₂→ 2 H₂O+ O₂

We have provided the first order reaction's reaction rate equation as,

                      Rate = [H2O2] k

As we are aware, the reaction rate equation is

                   Rate = rate constant × [H2O2] concentration

As a result, by comparing the two equations, we obtain, rate constant is k, and the [H2O2] concentration is 1 [H2O2].The concentration has doubled as a result. Afterward, the rate expression will be,

                           Rate = [H2O2] 2k.

As a result, we can infer that doubling the concentration will cause the reaction rate to double.

Learn more about how H2O2 breaks down here:

https://brainly.com/question/28047273

#SPJ1

A mass m is attached to an ideal massless spring. When this system is set in motion, it has a period t. What is the period if the mass is doubled to 2m?.

Answers

The period is √2 *T if the mass is doubled to 2m.

What is the time period?

The time period is the length of time during which an activity occurs or a condition remains.

The equation for the period T is given as:

[tex]T = 2\pi \sqrt{\frac{m}{k} }[/tex]

where k is the spring constant

mass of the system = m

Here,

If the mass id doubled to 2m for the system that is set in motion,

new mass is now  2m

So the equation for time period becomes,

[tex]T' = 2\pi \sqrt{\frac{2m}{k} }[/tex]

[tex]T' = \sqrt{2} * 2\pi \sqrt{\frac{m}{k} }[/tex]

[tex]T' = \sqrt{2}* T[/tex]

Hence,

The period is √2 *T if the mass is doubled to 2m.

Learn more about the time period here:

https://brainly.com/question/17188760

#SPJ4

The path of a charged particle moving parallel to a uniform magnetic field will be a:______.

Answers

The path of a charged particle moving parallel to a uniform magnetic field will be straight line

Force on a charged particle moving in a magnetic field can be calculated by = q.(v*B )

q = charge of the particle

v = velocity of the particle

B = magnetic field

theta = angle between v and B

q . (v B sin (theta) )

Since,  particle is parallel to the magnetic field

hence , theta = 0°

so sin(theta) = sin 0° = 0

hence , force = 0

since , there is no force acting on the particle it will remain in that motion in hat it was when it initially came in the magnetic field . Hence It will be moving along a straight line path because the magnetic force on the charged particle is zero.

To learn more about magnetic field here

https://brainly.com/question/17011493

#SPJ4

A car is being driven at a rate of 60 ft/sec when the brakes are applied. The car decelerates at a constant rate of 19

Answers

The car will take 300 m before it stops due to applying break.

What's the relation between initial velocity, final velocity, acceleration and distance?As per Newton's equation of motion, - = 2aSV= final velocity velocity of the object, U = initial velocity velocity of the object, a= acceleration, S = distance covered by the object Here, U = 60 ft/sec, V = 0 m/s, a= -6 ft/sec²So, 0² - 60² = 2×6× S

=> -3600 = -12S

=> S = 3600/12 = 300 m

Thus, we can conclude that the distance covered by the car is 300 m before it stopped.

Disclaimer: The question was given incomplete on the portal. Here is the complete question.

Question: A car is being driven at a rate of 60 ft/sec when the brakes are applied. The car decelerates at a constant rate of 6 ft/sec². How long will it take before the car stops?

Learn more about the Newton's equation of motion here:

brainly.com/question/8898885

#SPJ1

Patient is in the ed due to a football hitting his nose when playing tackle football in the park. X-ray shows a displaced nasal fracture. What icd-10-cm codes are reported?

Answers

ICD-10-CM codes are -S02.2XXA, W21.01XA, Y93.61, Y92.830


S02.2 for Fracture, Traumatic/Nasal (Bone(s)), ICD-10-CM Alphabetic Index. Both the open fracture code and the dislocation code are not reported. Only the fracture code is provided if a fracture and a dislocation happen at the same place. Search for "dislocation/with fracture" in the alphabetical index to be sent to a doctor. A closed fracture is a fracture with displacement. To report the conditions leading up to the injury, external cause codes are utilized. Look for Struck (accidentally) by/ball (struck) (thrown)/football W21.01 in the ICD-10-CM External Cause of Injuries Index. Seven characters are required in the Tabular List to finish the code. For the first encounter, X is utilized as a stand-in for character number six, and character number seven is given the letter A.

To learn more about ICD-10-CM please visit -https://brainly.com/question/27932590
#SPJ1

Frank and Lisa are analyzing the chart, which shows the speed at which light travels through different media.

Answers

Frank is correct because light travels through diamond at  the slowest speed, and speed and frequency have an inverse  relationship .

Lisa is correct because  light travels through air at the fastest speed of ,and speed and frequency have a direct relationship .

Neither  Frank nor Lisa  is correct because light  does not change frequency when it travels through different media .

Both Frank and Lisa are correct because light has both inverse and a direct relationship with frequency ,depending on the type of media .

Neither of the two is correct because light does not change frequency when it travels through different media .

Disclaimer :The question given on the portal is incomplete .Here is the complete question .

Question :Frank and Lisa are analyzing the chart, which shows the speed at which light travels through different media. Frank says that light travels through diamond with the greatest frequency. Lisa says that light travels through air with the greatest frequency. Which best describes the accuracy of these statements?

learn more from here: brainly.com/question/28098169

#SPJ4

A transformer usefully transfers 190,000 W of the 200,000 W of energy supplied to it. How efficient is the transformer?

Answers

Energy efficiency = useful energy output/ total energy input
Therefore (190000/200000) x 100 (to work out percentage) = 95%

A loop rests in the plane of a page of textbook while a magnetic field is directed into the page. A clockwise current is induced.

Answers

(b) when the magnetic field gets stronger.

correct option :

When a loop rest in the plane of the page and magnetic field is directed into the page then we will apply here right hand thumb rule in which thumb represent the direction of magnetic field and curling fingers represent direction of induced current. So, in this case the direction of magnetic field is into the page and curling fingers shows induced current in clockwise direction.

  Thus, option (b) is correct.

Incorrect options:

   option (A) is incorrect because when magnetic field will change  

     according to flux.

   option (C) is incorrect because field is stronger so it does not matter

     size of loop increases or decreases.

   option (D) is incorrect because it moves any other side the field will be

     stronger.

Learn more about magnetic field induction here:

 https://brainly.com/question/13610297

   #SPJ4

[Your question is incomplete, but most probably your full question was-Check all that apply:

A. when the magnetic field is tilted so it is no longer perpendicular to the page

B. when the magnetic field gets stronger

C. when the size of the loop decreases

D. when the loop is moved sideways across the page]

To lower the risk of a collision, you should keep at least __________ of space to one side of your vehicle at all times.

Answers

One must keep one car width of space to side of vehicle to avoid collision.

What is collision?

Collision is the sudden striking of two bodies that exerts forces on each other thus acelerating the objects in relatively very short time. One must maintain atleast one car width of space to side of vehicle to lower the risk of collision. If the space is lower than one car width than you should lower the speed of car to avoid collision.

One can also follow the two second disciple to avoid collision. According to this rule, the car should maintain two second safe distance behind any other vehicle .It acts as a safety buffer or defensive mechanism to avoid the risk of collision at all times at any speed. Also according to impulse momentum change theorem, the force on each object can be minimized by maintaing  more distance. As force and time are inversely proportional in collision.

Therefore, one car width distance is the safest minimum distance to avoid risks of collision .

Learn more about collision here:

https://brainly.com/question/14439547

#SPJ1

An ideal ammeter would have zero resistance while an ideal voltmeter would have infinite resistance, why?.

Answers

Answer: An ideal ammeter would have zero resistance, because to ensure that, there is no voltage drop due to the internal resistance. Similarly, an ideal voltmeter would have infinite resistance, because to ensure that there is no current is drawn by the voltmeter.

Explanation: To find the answer, we need to know about the Ammeter and Voltmeter.

What is an ammeter?An ammeter is a device, that can be used to measure the electric current flows through a circuit in amperes.An ideal ammeter would have zero resistance, because to ensure that, there is no voltage drop due to the internal resistance when it is connected in series to measure the current.What is voltmeter?A voltmeter is a device, that can be used to measure the electric potential difference generated between the terminals of an electric circuit in volts.An ideal voltmeter would have infinite resistance, because to ensure that there is no current is drawn by the voltmeter, when it is connected in parallel to measure the voltage.

Thus, we can conclude that, an ideal ammeter would have zero resistance, because to ensure that, there is no voltage drop due to the internal resistance. Similarly, an ideal voltmeter would have infinite resistance, because to ensure that there is no current is drawn by the voltmeter.

Learn more about the ammeter and voltmeter here:

https://brainly.com/question/28044897

#SPJ4

A 67 V power supply is connected to three identical resistors in series. If the current through the resistors is measured at 0.95 A, what is the value of each resistor? Round your answer to 2 decimal places.

Answers

The value of the identical resistors would be 70. 53 Ohms

How to determine the values

We have the formula for resistance to be;

Resistance = Voltage/ current

Voltage = 67 V

Current = 0. 95A

Substitute the values

Resistance =

[tex] \frac{67}{0.95} [/tex]

Resistance = 70.53 Ohms

Thus, the value of the identical resistors would be 70. 53 Ohms

Learn more about resistance here:

https://brainly.com/question/14883923

#SPJ1

During World War II, the United States government forced thousands of _____ to be relocated to internment camps.

Answers

Answer:

Japanese Americans

Explanation:

prevented them from buying land and sent them back to japan after the war

hope this helps :)

Answer:

Japanese Americans

Explanation:

Since Japan was helping the Axis, the United States were afraid that if Japan attacked the Continental United States, then Japanese Americans would help them.

This fear led to the government placing Japanese Americans in internment camps.

Select the correct answers. which two pieces of information show that a research source is reliable?\

Answers

Answer:

2

Explanation:

just took the assignment

Rolls of aluminum foil are 304-mm wide and 0.016-mm thick. what is the maximum length of aluminum foil that can be made from 1.10-kg of aluminum, al? density of al = 2.70 g/cm3

Answers

Answer:

See below

Explanation:

The units in this question need to be all the same

Volume = L x W x H      where 'H' is thickness of the foil

1.10 kg = 1100 g

The volume given =   mass / density  =  1100 g / 2.70 g/cm^3=  407.407  cm^3

304 mm = 30.4 cm    .016 mm = .0016 cm

LWH = L * 30.4 cm  * .0016 cm    = 407.407 cm^3

L = 8375.97 cm     = 83.76 meters

The maximum length of aluminum foil that can be made from 1.10-kg of aluminum would be 87.76 meters.

What is density?

It can be defined as the mass of any object or body per unit volume of the particular object or body. Generally, it is expressed as in gram per cm³ or kilogram per meter³.

By using the above formula for density

ρ = mass / volume

As given in the problem Rolls of aluminum foil are 304 mm wide and 0.016-mm thick.

the mass of the aluminum =  1.10-kg

mass = 1100 grams

the density of the aluminum = 2.70 g/cm3

density = 17/3

2.70 g/cm3 = 1100 grams /volume

volume = 1100/ 2.70

             =407.40 cm3

volume = width×thickness×length

407.40 cm3  = 30.4 ×.0016 ×length

length = 8375.97 cm

           = 87.76 meters

Thus, the maximum length of aluminum foil that can be made from 1.10-kg of aluminum would be 87.76 meters.

To learn more about density from here, refer to the link;

brainly.com/question/15164682

#SPJ2

A tank, shaped like a cone has height 6 meters and base radius 5 meters long. It is placed so that the circular part is upward. It is full of water, and we have to pump it all out by a pipe that is always leveled at the surface of the water. Assume that a cubic meter of water weighs 10 000 N , i.e. the density of water is 10000 N m 3 . How much work does it require to pump all water out of the tank

Answers

The work required to pump out the liquid is 2.35 MJ.

What is work?

Work can be defined as the scalar product of the force and the displacement.

The formula to calculate the work done W is,

[tex]W=F.d[/tex]

where F is the force and d is the displacement.

Note: It is assumed that the weight of one cubic unit of the liquid is [tex]10000 \text{ N/m}^3[/tex].

Break down the vertical tank into parallel disc elements whose thickness is dh and are located at a height h from the ground. The radius of the element at height h is r.

It is given that the height of the tank is 6 m and the base radius is 5 meters. Using these, the ratio of the radius r at height h to the height h by using the triangle similarity criterion is,

[tex]\frac{h}{r}=\frac{6}{5} \\ r=\frac{5h}{6}[/tex]

Using the above value of r, the volume of the element is given by,

[tex]V= \pi r^2dh\\V=\pi (\frac{5h}{6})^2dh\\V= \frac{25\pi h^2}{36} dh[/tex]

The weight of the volume element is the force required.

The weight of one cubic unit of the liquid is [tex]10000 \text{ N/m}^3[/tex].

So the weight F of the volume element at height h is given by,

[tex]F=10000 (\frac{25 \pi h^2}{ 36})dh\\F= 6944.44\pi h^2dh[/tex]

At the height h, the difference in the height of the top of the tank and the volume element is (6-h). So the work required to pump out the volume of liquid at height h is given by the integrating the equation,

[tex]W=6944.44\pi\int\limits^6_0 {(6h^2-h^3)} \, dx \\\\W=69.44\pi \left[ 2h^3-\frac{h^4}{4}\right]^6_0\\W=2,.35\times10^6\text{ J}[/tex]

The value of the W obtained is 2.35*10^6 J or 2.35 MJ.

Learn more about work here:

https://brainly.com/question/16627899

#SPJ4

A multidimensional database (MDDB) is a type of database in which data can be viewed from multiple dimensions; commonly used with ____.

Answers

A multidimensional database (MDDB) is a type of database in which data can be viewed from multiple dimensions; commonly used with MOLAP.

A multidimensional database offers the capacity to swiftly process records and generates answers quickly. MDBs allow customers ask questions about agencies' operations and traits.

The multidimensional databases employ MOLAP (multidimensional on line analytical processing) to get entry to its records. They allow the customers to speedy get solutions to their requests through producing and studying the information as a substitute speedy. The information in multidimensional databases is saved in a statistics dice layout.

In a dimensional database, you've got rows and columns, represented by way of X and Y. In a multidimensional database, you've got were given X, Y, Z, etc. depending on the number of dimensions to your statistics. below is an example of a 3-Dimensional information Array represented in a relational desk and in three-D.

Learn more about the multidimensional databases here: https://brainly.com/question/518894

#SPJ1

Answer the following regarding Momentum Change and Force

1. Find the force necessary to gain a velocity of 60 m s-1 within 5 minutes, when acted on a mass of 200 kg which is initially at rest.

2. A force of 150 N acts on an object of mass 600 kg for 2 minutes, which was initially at rest. Find the velocity gained by the object at the end of the second minute.

Answers

The force necessary to gain the velocity is 40 N.

The velocity gained by the object at the end of the second minute is 30 m/s.

Force of the object

F = mv/t

where;

m is mass of the object = 200 kgv is velocity of the object = 60 m/st is time of motion = 5 mins = 300 seconds

F = (200 x 60)/(300)

F = 40 N

Velocity of the object

F = mv/t

Ft = mv

v = Ft/m

where;

m is mass = 600 kgF is force = 150 Nt is time = 2 mins = 120 s

v = (150 x 120)/(600)

v = 30 m/s

Thus, the force necessary to gain the velocity is 40 N.

The velocity gained by the object at the end of the second minute is 30 m/s.

Learn more about force here: https://brainly.com/question/12970081

#SPJ1

A 75.0-kg man standing on a bathroom scale in an elevator. Calculate the scale in N, reading if the elevator moves upward at a constant speed of 1.5 m/s

Answers

A 75.0-kg man standing on a bathroom scale in an elevator, if the elevator moves upward at a constant speed of 1.5 m/s then the scale in N will be 885N accordance with Newton's law

 

The extent of the force being applied downward. If the human is selected to be the system of interest with a free-body diagram, the one-dimensional problem appears to be considerably more difficult. Both sections (a) and (b) of this example's analysis can be answered by analyzing the free-body diagram in accordance with Newton's laws, along with some other potential problems. In order to determine what the scale reads, the only forces operating on the subject are his weight w and the upward force of the scale, which are equal in strength and directed in opposing directions. Newton's law, as usual, can be used to do this.

F=Ma  = F−w=ma⇒F=m(a+g)=75(2+9.8)=885 N​.


To learn more about such problems on Newton's law please visit -
https://brainly.com/question/8052658
#SPJ4

Two hockey pucks with mass 0.1 kg slide across the ice and collide. Before
the collision, puck 1 is going 15 m/s to the east and puck 2 is going 12 m/s to
the west. After the collision, puck 2 is going 15 m/s to the east. What is the
velocity of puck 1?
A. 12 m/s west
B. 15 m/s east
C. 15 m/s west
D. 12 m/s east

Answers

The velocity of pluck 1 is 12 m/s west.

What is the conservation of momentum?

The principle of the conservation of the linear momentum states that momentum before collision is equal to momentum after collision.

Now given that;

m1u1 + m2u2 = m1v1 + m2v2

(0.1 * 15) - (0.1 * 12) = 0.1* v + (0.1 * 15)

1.5 - 1.2 = 0.1v + 1.5

0.3 - 1.5 = 0.1v

v = -1.2/0.1

v = - 12 m/s

Hence, the velocity of pluck 1 is 12 m/s west.

Learn  more about linear momentum:https://brainly.com/question/27988315

#SPJ1

When driving on a highway, sudden strong cross wind gusts: Select one: a. Do not affect a car as much as a strong head wind b. Affect large cars more than small cars c. Always cause severe dust problems d. Can move a car sideways into another lane

Answers

When driving on a highway, sudden strong cross wind gusts move a car sideways into another lane (option D).

What is a crosswind?

A crosswind is a wind blowing across a line of travel.

Accidents have been reported to be caused by strong winds and can cause heavy distractions that will cause the driver to lose control.

However, a strong and sudden crosswind will cause the car to move sideways into another lane, and can lead to severe accident.

Learn more about crosswind at: https://brainly.com/question/2814024

#SPJ1

A car increases its speed from 30 km/h to 60 km/h in 5 seconds. Its acceleration​

Answers

Answer:6km/sec

Explanation: Acceleration = Change in velocity / Time

Change in velocity = 60km/h minus 30km/hr

Time = 5seconds

Other Questions
Question 8Match each compound to its correct type.8.1COhydroxide8.2COcarbonate8.3SO8.4CaCO8.5NaOHoxidetrioxidedioxide FILL IN THE BLANK. A(n)____ is a small table consisting only of a list of the primary key field foreach record in a table along with location information for that record. use the table to evaluate each expression. x 1 2 3 4 5 6 f(x) 1 4 3 4 1 1 g(x) 4 5 2 3 4 3 (a) f(g(1)) (b) g(f(1)) (c) f(f(1)) (d) g(g(1)) (e) (g f)(3) (f) (f g)(6) A beam of unpolarized light in material X, with index 1.19, is incident on material Y. Brewster's angle for this interface is found to be 46.3 degrees. What is the index of refraction of material Y?a) 1.60 b) 1.25 c) 0.976 d) 1.40 Which of the following might contribute to psychogenic impotence? A. Disease in any other body system. B. Injury to pelvic organs. C. Medication why using an aquatic plant and measuring oxygen is the easiest way to measure the ""output"" of photosynthesis. In a simple random sample of size 98, there were 37 individuals in the category of interest. Compute the sample proportion p. O 0.378 0.622 O 0.607 135 An envelope is 4 cm longer than it is wide the area is 36 cm find the length width an electron traveling at a speed of 5.80 x 10^6 strikes the target of an x ray tbe . Upon impact, the electron decelerates to two-third of its original speed, with an X-ray photon being emitted in the process. What is the wavelength of the photon? when firms in monopolistic competition earn positive economic profits, other firms tend to In a volcano, erupting lava flows continuously through a tube system about 14 kilometers to the sea. Assume a lava flow speed of 0.5 kilometer per hour and calculate how long it takes to reach the sea. t takes hours to reach the sea. (Type an integer or a decimal.) michael kimathi has purchased a tractor for $95,000. he expects to receive a net cash flow of $28,750 per year from the investment. what is the payback period for michael? The gas phase decomposition of dimethyl ether at 500 C CH3OCH3(g) CH4(g) + H2(g) + CO(g) is first order in CH3OCH3 with a rate constant of 4.0010-4 s-1. If the initial concentration of CH3OCH3 is 3.4810-2 M, the concentration of CH3OCH3 will be M after 5158 s have passed. What percentage of the area under the normal curve is to the left of z1 and to the right of z2? Round your answer to two decimal places.z1=1.50z2=0.39 use mathematical induction to show that 2n > n2 n whenever n is an integer greater than 4. printing money levying a sales tax making a treaty with Canada creating the high school curriculum managing elections Federal powers or State powers Use the Gram-Schmidt process to find an orthonormal basis for the subspace of R4 spanned by the vectors u1 = (1; 0; 0; 0); u2 = (1; 1; 0; 0); u3 = (0; 1; 1; 1): Show all your work. a sample currently contains 20 parent atoms and 60 daughter atoms. given that the half-life is 2,000 years how old is the sample? A linear equation with a slope of -3 is steeper or less steep than one with a slope of -5 T/F : what prints? int a[] = {1, 3, 5, 7, 9}; int *p = a; cout